Challenging Integrals in Calculus 1-2: Expand Your Problem-Solving Skills!

  • Thread starter Thread starter pakmingki
  • Start date Start date
  • Tags Tags
    Hard Integrals
  • #51
Hii,

I found that this topic about calculus is really interesting..
I suggest that we could continue this topic, of course with some new problems. Besides, anyone who could do the integration should put the solution, except the one who purpose the question itself. So, we can improve our skill together. How ?? I hope that everyone have the same thought. Hehe.
Thanks.
 
Physics news on Phys.org
  • #53
yip said:
For that integral, i think u have to use the partial fraction theorem that involves derivatives, as there is trigonometric terms in the primitive of that function. Here is how I did the second integral, since u asked, the trick is to apply euler's theorem:

e^{-ix^{2}}=cosx^{2}-isinx^{2}
\int_{0}^{\infty}e^{-ix^{2}}dx=\int_{0}^{\infty}cosx^{2}dx-i\int_{0}^{\infty}sinx^{2}dx
It is known that \int_{0}^{\infty}e^{-x^{2}}dx=\frac{\sqrt{\pi}}{2}
\int_{0}^{\infty}e^{-ix^{2}}dx=\frac{\sqrt{\pi}}{2\sqrt{i}}
\frac{1}{\sqrt{i}}=\frac{1-i}{\sqrt{2}}
\int_{0}^{\infty}e^{-ix^{2}}dx=\frac{1}{2}\sqrt{\frac{\pi}{2}}(1-i)
\int_{0}^{\infty}sinx^{2}dx=I\int_{0}^{\infty}e^{-ix^{2}}dx=\frac{1}{2}\sqrt{\frac{\pi}{2}}

This is oversimplifying the proof. The usual trick where gaussian integral is calculated as a square root of the two dimensional gaussian integral, gives the equation
<br /> \int\limits_{-\infty}^{\infty} e^{-(A+iB)x^2}dx = \sqrt{\frac{\pi}{A+iB}}<br />
in which A and B are real, only when A &gt; 0. (The square root is the one that is continuous on set \{z\in\mathbb{C}\;|\; \textrm{Re}(z)&gt;0\}, and that agrees with the positive square root on real axis.) If A=0, then the substitution r=\infty does not converge in the two dimensional integral, and thus the result is not trivially the same.

However, the Leibniz's rule of alternating series can be used to show that integrals
<br /> \int\limits_{0}^{\infty}\cos(x^2)dx<br />
and
<br /> \int\limits_{0}^{\infty}\sin(x^2)dx<br />
converge towards something, and thus the integral
<br /> \int\limits_{-\infty}^{\infty} e^{ix^2}dx<br />
also exists. If we know that the integral is a continuous function of the coefficent in the exponent, then this is all done, but I never bothered actually try to figure out how the continuity could be proven. If it can be done nicely, feel free to mention how.
 
Last edited:
  • #54
jostpuur said:
However, the Leibniz's rule of alternating series can be used to show that integrals
<br /> \int\limits_{0}^{\infty}\cos(x^2)dx<br />
and
<br /> \int\limits_{0}^{\infty}\sin(x^2)dx<br />
converge towards something, and thus the integral
<br /> \int\limits_{-\infty}^{\infty} e^{ix^2}dx<br />
also exists. If we know that the integral is a continuous function of the coefficent in the exponent, then this is all done, but I never bothered actually try to figure out how the continuity could be proven. If it can be done nicely, feel free to mention how.

In fact it is known what they converge http://www.artofproblemsolving.com/Forum/viewtopic.php?t=158083"
 
Last edited by a moderator:
  • #55
The approach that starts with Gaussian integrals leads to the question about limit of the expression
<br /> \int\limits_{-\infty}^{\infty} e^{-Ax^2}dx<br />
when A approaches some value on the line \{z\in\mathbb{C}\;|\; \textrm{Re}(z)=0\} from right.

I admit, I hadn't seen the gamma function approach earlier, and it seems to avoid this problem.
 
  • #56
Or actually I'm not so sure about this all yet. The derivation that uses gamma functions seems to assume right in the beginning that integral
<br /> \int_{0}^{\infty} e^{ix^k} dx<br />
converges. It would be nice to show it some Leibniz's test, for convergence, before continuing with variable changes, that are used for solution of the value of the integral.
 
  • #57
jostpuur said:
Or actually I'm not so sure about this all yet. The derivation that uses gamma functions seems to assume right in the beginning that integral
<br /> \int_{0}^{\infty} e^{ix^k} dx<br />
converges. It would be nice to show it some Leibniz's test, for convergence, before continuing with variable changes, that are used for solution of the value of the integral.

I believe convergence exists when k>1.
 
  • #58
JohnDuck said:
I'm stumped but intrigued.
Isn't it just 0?
 
  • #59
\Gamma(z)=\underset{0}{\overset{\infty}{\int}}\gamma^{z-1}e^{-\gamma}d\gamma

Make like a snob and use the gamma-function.
 
  • #60
\int {sinxcosx \over sin^4x+cos^4x}dx
 
  • #61
camilus said:
I =\int {sinxcosx \over sin^4x+cos^4x}dx

Is that supposed to be difficult, or what ?

I= -\frac{1}{2}\int \frac{d(\cos 2x)}{(\cos 2x)^2 + 2\left[1-(\cos 2x)^2\right]} =...

ends up in something proportional to argth(fraction involving arccos).
 
  • #62
double angle identity for sine allowed me to solve in terms of arctangent when u=cos(2x)
 
  • #63
Did you get arctangent circular or arctangent hyperbolic ?
 
  • #64
circular
 
  • #65
it's wrong, because there's a minus in the denominator. it should be the hyperbolic artangent function.
 
  • #66
i doubt it. Even wolfram integrator says its correct. Show your work?
 
  • #67
here is a sexy one

\\int \\sqrt {\\x^2+1} dx whoops latex is rusty ... be patient dang how thu??
 
  • #68
You can use tex tags to display \LaTeX.

For the integral, do a trig sub.
 
  • #70
This one can be done without the hyperbolic functions but it is a good page long

\int \sqrt {x^2 + 1}\, dx
 
Last edited:
  • #71
\int\!\!\sqrt{x^2+1}\,\mathrm{d}x is a fun one...
 
  • #72
\int\!\!\sqrt{x^2+1}\,\mathrm{d}x looks like a problem I'd be banging my head against the wall to solve. I'm assuming you use tan2 + 1 = sec2 and some funny manipulation?
 
  • #73
Ok, people are accustomed to solve the \int \sqrt{x^2 +1} \, dx by some trigonometric substitution, either hyperbolic sine, or circular secant/cosecant.

But there's a third way which is not transcendental until the end. The substitution

\sqrt{x^2 +1} - x = t
 
  • #74
(tanx)^1/2
 
  • #75
andyb177 said:
(tanx)^1/2

\int \sqrt{\tan x} \, dx

is a well-known elliptic integral. It can be evaluated by Mathematica explicitely.
 
  • #76
gb7nash said:
\int\!\!\sqrt{x^2+1}\,\mathrm{d}x looks like a problem I'd be banging my head against the wall to solve. I'm assuming you use tan2 + 1 = sec2 and some funny manipulation?

Why couldn't you just perform a general u-substitution with this setting u = x^2 + 1 du =2xdx, dx= du/2x -> int [sqrt(u)/2x]du x=+/-sqrt(u-1)
lol, nevermind. Sorry!
 
  • #77
If we let u^2 = \tan x we have
\[ \int \frac{ 2u^2}{1+ u^4} du = \int \frac{ u^2 + 1}{1+u^4} du + \int \frac{ u^2 - 1}{1+u^4} du \]<br /> <br /> \[ = \int \frac{ 1+ \frac{1}{u^2} }{u^2 + \frac{1}{u^2} } du + \int \frac{ 1- \frac{1}{u^2} }{u^2 + \frac{1}{u^2} } du \]<br /> <br /> \[ = \int \frac{ d\left( u - \frac{1}{u} \right) }{ \left( u - \frac{1}{u} \right)^2 +2 } + \int \frac{ d\left( u + \frac{1}{u} \right) }{ \left( u + \frac{1}{u} \right)^2 -2 } \]<br /> <br /> \[ = \frac{ 1}{\sqrt{2}} \left( \tan^{-1} \left( \frac{ u - \frac{1}{u}}{\sqrt{2}} \right) - \tanh^{-1} \left( \frac{ u + \frac{1}{u}}{\sqrt{2}} \right) \right) + C \]<br /> <br /> \[ = \frac{ 1}{\sqrt{2}} \left( \tan^{-1} \left( \frac{ \sqrt{\tan x} - \sqrt{\cot x}}{\sqrt{2}} \right) - \tanh^{-1} \left( \frac{ \sqrt{\tan x} + \sqrt{\cot x}}{\sqrt{2}} \right) \right) + C \]
 
  • #78
Would anyone be interested if I started a new topic here, and posted all the integrals I have ?

I have spent perhaps a week going through my book and various webpages to find all the challenging and interesting integrals I could. Some are taken from here but many are not.

I think I have about 100-120 integrals. Ranging from easy to really hard. Would anyone be interested in that? Ofcourse I could post all of them here, but it would be messy. Much nicer with a first post containing integrals.
 
  • #79
Sure why not. I'll even attempt to do them using complex analysis. Maybe.
 
  • #80
\int xe^{ax}sin(bx) dx

I LOVED solving this one; took an intellectual pleasure in it, to be honest :P .

It's not hard actually, just quite a bit of work.
 
Last edited:
  • #81
you guys really love your taylor expansions
 
  • #82
Check this

:biggrin:

∫1/(1+x4) dx
 

Attachments

  • inter.jpg
    inter.jpg
    3.1 KB · Views: 956
  • #83
Here is an interesting one I came up with.
\int_{-\frac{\pi}{2}}^{\frac{\pi}{2}}\frac{\log(\cos(x))dx}{e^x+1}
 
Last edited:
  • #84
Please give me an answer!

see the question_attached and please let me know your method to solve this. :)
 

Attachments

  • integrl_ln.jpg
    integrl_ln.jpg
    3.9 KB · Views: 976
Last edited:
  • #85
First you can drop the absolute value for the sine, since it's completely positive on the integration domain. Then I'd use a trick writing

\sin x = \mbox{Im}\left(e^{ix}\right).
 
  • #86
dextercioby said:
First you can drop the absolute value for the sine, since it's completely positive on the integration domain. Then I'd use a trick writing

\sin x = \mbox{Im}\left(e^{ix}\right).

That method doesn't work since it arrives at a different result than the correct answer. The imaginary part cannot be taken after the integral to obtain the same answer since the logarithm doesn't work that way.
 
Last edited:
  • #87
chandi2398 said:
see the question_attached and please let me know your method to solve this. :)

The correct way to evaluate this integral is as follows. Call the integral ##\displaystyle I##.

Sub ##\displaystyle y = \frac{\pi}{2} - x##.

Now you can prove that ##\displaystyle I = \int_0^\frac{\pi}{2} \ln \cos y dy = \int_0^\frac{\pi}{2} \ln \cos x dx##.

Hence ##\displaystyle 2I = \int_0^\frac{\pi}{2} \ln (\frac{1}{2}\sin 2x) dx = \int_0^\frac{\pi}{2}(-\ln 2)dx + \int_0^\frac{\pi}{2}\ln \sin 2x dx = -\frac{\pi\ln 2}{2} + \int_0^\frac{\pi}{2}\ln \sin 2x dx ##.

Now ##\displaystyle \int_0^\frac{\pi}{2}\ln \sin 2x dx = \frac{1}{2}\int_0^{\pi}\ln \sin x dx## as should become apparent after another sub of ##\displaystyle u = 2x##.

##\displaystyle \frac{1}{2}\int_0^{\pi}\ln \sin x dx = (2)(\frac{1}{2})\int_0^{\frac{\pi}{2}}\ln \sin x dx = I ## by symmetry.

So you're left with ##\displaystyle 2I = I -\frac{\pi\ln 2}{2}## yielding ##\displaystyle I = -\frac{\pi\ln 2}{2}##
 
  • #88
3 very hard ones (spoiler alert: they do not have an anti derivative)

\int^1_0 \frac{\ln(1+x)}{1+x^2} dx


\int^1_0 \frac{x-1}{\ln(x)} dx

\int^1_0 \frac{\ln(1-x)}{x} dx
 
  • #89
dumbperson said:
\int^1_0 \frac{\ln(1+x)}{1+x^2} dx
This one is fairly straightforward, use the substitution ##x=\tan\theta##.
\int^1_0 \frac{x-1}{\ln(x)} dx
For this one, define:
$$I(a)=\int_0^1 \frac{x^a-1}{\ln(x)}dx$$
Differentiate both the side with respect to a to get:
$$\frac{dI}{da}=\int_0^1 x^a\,dx=\frac{1}{a+1}$$
$$\Rightarrow I(a)=\ln|a+1|+C$$
It can be easily seen that C=0. We need the value of I(a) when a=1, hence,
$$I(1)=\ln(2)$$
\int^1_0 \frac{\ln(1-x)}{x} dx
We use the series expansion of ##\ln(1-x)## i.e
$$\ln(1-x)=-\sum_{k=1}^{\infty} \frac{x^k}{k}$$
Hence, our integral is:
$$-\int_0^1 \frac{1}{x}\sum_{k=1}^{\infty} \frac{x^k}{k}\,dx$$
$$=-\sum_{k=1}^{\infty} \int_0^1 \frac{x^{k-1}}{k}\,dx$$
$$=-\sum_{k=1}^{\infty} \frac{1}{k^2}=-\zeta(2)$$
 
  • #90
Goa'uld said:
Here is an interesting one I came up with.
\int_{-\frac{\pi}{2}}^{\frac{\pi}{2}}\frac{\log(\cos(x))dx}{e^x+1}

Let
$$I=\int_{-\frac{\pi}{2}}^{\frac{\pi}{2}}\frac{\log(\cos(x))dx}{e^x+1}$$
We can also write:
$$I=\int_{-\frac{\pi}{2}}^{\frac{\pi}{2}}\frac{\log(\cos(x))dx}{e^{-x}+1}$$
Add both the expressions for I to get:
$$2I=\int_{-\pi/2}^{\pi/2} ln(\cos(x))\,dx=2\int_0^{\pi/2}ln(\cos(x))\,dx$$
I can rewrite the above as:
$$I=\int_0^{\pi/2} \ln(\sin(x))$$
The above definite integral is evaluated by Curious3141 in his post #87.
 
  • #91
Pranav-Arora said:
This one is fairly straightforward, use the substitution ##x=\tan\theta##.

For this one, define:
$$I(a)=\int_0^1 \frac{x^a-1}{\ln(x)}dx$$
Differentiate both the side with respect to a to get:
$$\frac{dI}{da}=\int_0^1 x^a\,dx=\frac{1}{a+1}$$
$$\Rightarrow I(a)=\ln|a+1|+C$$
It can be easily seen that C=0. We need the value of I(a) when a=1, hence,
$$I(1)=\ln(2)$$

We use the series expansion of ##\ln(1-x)## i.e
$$\ln(1-x)=-\sum_{k=1}^{\infty} \frac{x^k}{k}$$
Hence, our integral is:
$$-\int_0^1 \frac{1}{x}\sum_{k=1}^{\infty} \frac{x^k}{k}\,dx$$
$$=-\sum_{k=1}^{\infty} \int_0^1 \frac{x^{k-1}}{k}\,dx$$
$$=-\sum_{k=1}^{\infty} \frac{1}{k^2}=-\zeta(2)$$

Are you sure about the tan substitution? I couldn't solve it that way, but I'm not very good
 
  • #92
dumbperson said:
Are you sure about the tan substitution? I couldn't solve it that way, but I'm not very good

Hi dumbperson! :)

Yes, I am sure about it. After the substitution, you should get:
$$I=\int_0^{\pi/4} \ln(1+\tan\theta)\,d\theta$$
Also,
$$I=\int_0^{\pi/4} \ln\left(1+\tan\left(\frac{\pi}{4}-\theta\right)\right)\,d\theta=\int_0^{\pi/4} \ln\left(1+\frac{1-\tan\theta}{1+\tan\theta}\right)\,d\theta=\int_0^{\pi/4} \ln(2)-\ln(1+\tan\theta) \,d\theta$$
$$\Rightarrow I=\frac{\pi \ln2}{4}-I \Rightarrow \boxed{I=\frac{\pi \ln2}{8}}$$

I hope that helps.
 
  • #93
Pranav-Arora said:
Hi dumbperson! :)

Yes, I am sure about it. After the substitution, you should get:
$$I=\int_0^{\pi/4} \ln(1+\tan\theta)\,d\theta$$
Also,
$$I=\int_0^{\pi/4} \ln\left(1+\tan\left(\frac{\pi}{4}-\theta\right)\right)\,d\theta=\int_0^{\pi/4} \ln\left(1+\frac{1-\tan\theta}{1+\tan\theta}\right)\,d\theta=\int_0^{\pi/4} \ln(2)-\ln(1+\tan\theta) \,d\theta$$
$$\Rightarrow I=\frac{\pi \ln2}{4}-I \Rightarrow \boxed{I=\frac{\pi \ln2}{8}}$$

I hope that helps.

Ah cool, thanks!
 
  • #94
Not really a hard one but try this: \displaystyle\int 1-\sin x\cos x\tan x\,dx
 
  • #95
Some of my most memorable ones from calc I, should take a few coffees to solve:

$$\int_{0}^{∞} \frac{1}{x^2 + \sqrt{x}} dx$$

$$\int_{0}^{\frac{\pi}{4}} \frac{1}{1 + sin^2(x)} dx$$
 
  • #96
Zondrina said:
$$\int_{0}^{\frac{\pi}{4}} \frac{1}{1 + sin^2(x)} dx$$

if we divide both the numerator and the denominator by \frac{1}{cos^{2}x} the integral becomes

\displaystyle\int_{0}^{\frac{\pi}{4}} \frac{sec^{2}x}{1+2tan^{2}x} dx

We can make the substitution t = tan(x)

I guess you can solve the first one by making the sub u = \sqrt{x} and then using partial fractions decomposition
 
Last edited:
  • #97
This one was a 1968 Putnam competition problem, I believe:
{\int_{0}^{1}{\frac{x^4 (1-x)^4}{1+x^2} dx}}
The answer is really interesting...
If you're really up for a challenge, try this continuation:
{\int_{0}^{1}{\frac{x^8 (1-x)^8 (25+816x^2)}{3164(1+x^2)} dx}}
These are just tedious and definitely easier than many of the problems here.
-- Joseph
 
  • #98
The 1st one is not tedious:

- \int_{0}^{1} \frac{(1-x^4 -1)(1-x)^4}{1+x^2} dx = - \int_{0}^{1} (1-x)^5 (1+x) dx + \int_{0}^{1} \frac{(1-x)^4}{1+x^2} dx

For the 1st integral, just sub 1-x = p and it will be trivial.

For the 2nd integral, write the integrand as

\frac{(1-x)^2 (1-x)^2}{(1-x)^2 + 2x} = (1-x)^2 - \frac{2x(1-x)^2}{(1-x)^2+2x} = (1-x)^2 - 2x\frac{1-x^2}{1+x^2} + \frac{4(1+x^2)-4}{1+x^2}

The integration of the 4 terms is elementary.
 
  • #99
dextercioby said:
The 1st one is not tedious:

- \int_{0}^{1} \frac{(1-x^4 -1)(1-x)^4}{1+x^2} dx = - \int_{0}^{1} (1-x)^5 (1+x) dx + \int_{0}^{1} \frac{(1-x)^4}{1+x^2} dx

Can you explain in more detail exactly what you did there please? I see that ##x^4 = -(-x^4 +1 - 1)## and then I don't see how you got to the next step.

I had a good time with this one:

\int_{\to 0}^{\to 1}\ln x \ln (1-x) \, \mathrm dx

(+7 points if you prove that it exists before evaluating it)
 
  • #100
GFauxPas said:
I had a good time with this one:

\int_{\to 0}^{\to 1}\ln x \ln (1-x) \, \mathrm dx

(+7 points if you prove that it exists before evaluating it)

$$\int_0^1 \ln x\ln(1-x)\,dx=-\sum_{k=1}^{\infty} \frac{1}{k}\int_0^1 x^k\ln x\,dx=\sum_{k=1}^{\infty} \frac{1}{k(k+1)^2}$$
$$=\sum_{k=1}^{\infty} \frac{1+k-k}{k(k+1)^2}=\sum_{k=1}^{\infty} \frac{1}{k(k+1)}-\sum_{k=1}^{\infty} \frac{1}{(k+1)^2}$$
Both the sums are easy to evaluate, the first one evaluates to 1 by telescoping series and the second one is simply ##\zeta(2)-1##, hence, the final answer is ##2-\zeta(2)##.

einstein314 said:
This one was a 1968 Putnam competition problem, I believe:
{\int_{0}^{1}{\frac{x^4 (1-x)^4}{1+x^2} dx}}
The answer is really interesting...
If you're really up for a challenge, try this continuation:
{\int_{0}^{1}{\frac{x^8 (1-x)^8 (25+816x^2)}{3164(1+x^2)} dx}}
These are just tedious and definitely easier than many of the problems here.
-- Joseph

http://en.wikipedia.org/wiki/Proof_that_22/7_exceeds_π
 
  • Like
Likes dextercioby
Back
Top